LSAT and Law School Admissions Forum

Get expert LSAT preparation and law school admissions advice from PowerScore Test Preparation.

User avatar
 Dave Killoran
PowerScore Staff
  • PowerScore Staff
  • Posts: 5849
  • Joined: Mar 25, 2011
|
#82646
Complete Question Explanation
(The complete setup for this game can be found here: lsat/viewtopic.php?t=1025)

The correct answer choice is (A).

This is a Suspension question, but, it is also a List question and so it is less time-consuming than other Suspension questions. In general, if you have time problems, avoiding a Suspension question is a reasonable strategy. But, do not avoid Suspension List questions as they can be solved as quickly as normal List questions!

This time Law Services is very clever, for although one of the rules is suspended, of course the third rule (“Ranch has a Tudor next to it”) is still in force and this proves answer choice (A) correct in this Except question. Remember, this is still a List question and proper List question technique applies.
 ilovethelsat
  • Posts: 1
  • Joined: Jul 20, 2020
|
#77325
If the condition requiring house 6 to be a split-level house is suspended, what deductions could be made which affect houses 2,4,6, and 8?
If I have the following options:
1: T
3: R
5: S
7: T
or
1: T/S
3: R
5: T
7: S/R

How would I know with certainty what would not be able to go in slots 2, 4, 6, and 8? The only option I was able to get rid of was answer choice A because there was a Ranch that was not next to a Tudor.
 Rachael Wilkenfeld
PowerScore Staff
  • PowerScore Staff
  • Posts: 1358
  • Joined: Dec 15, 2011
|
#77359
Hi ilovethelsat,

This is a suspension question, so we want to think about what rules we have left after suspending the rule about house 6. We still have R in 3, but that doesn't have any impact on 2, 4, 6, 8. So all we have to do is comply with the rules that adjacent houses can't be the same style, and that any R has to have at least one T adjacent. We aren't going to write all the possible arrangements, but instead, we are going to look for an answer that breaks one of those two rules. We don't see any two adjacent houses that are the same style in the answers, but we do have an R in answer choice (A) that is not next to any Ts. That is why we can select that answer choice, and move on.

As an aside, if you see a cannot be true question in the games, don't start by trying to think of ALL the things that could be true. It will slow you down. Focus on figuring out what things cannot be true, and look for that first.

Hope that helps!
Rachael
 Rachael Wilkenfeld
PowerScore Staff
  • PowerScore Staff
  • Posts: 1358
  • Joined: Dec 15, 2011
|
#77360
Hi ilovethelsat,

This is a suspension question, so we want to think about what rules we have left after suspending the rule about house 6. We still have R in 3, but that doesn't have any impact on 2, 4, 6, 8. So all we have to do is comply with the rules that adjacent houses can't be the same style, and that any R has to have at least one T adjacent. We aren't going to write all the possible arrangements, but instead, we are going to look for an answer that breaks one of those two rules. We don't see any two adjacent houses that are the same style in the answers, but we do have an R in answer choice (A) that is not next to any Ts. That is why we can select that answer choice, and move on.

As an aside, if you see a cannot be true question in the games, don't start by trying to think of ALL the things that could be true. It will slow you down. Focus on figuring out what things cannot be true, and look for that first.

Hope that helps!
Rachael

Get the most out of your LSAT Prep Plus subscription.

Analyze and track your performance with our Testing and Analytics Package.